LSAT 15 – Section 3 – Question 07

You need a full course to see this video. Enroll now and get started in less than a minute.

Target time: 1:17

This is question data from the 7Sage LSAT Scorer. You can score your LSATs, track your results, and analyze your performance with pretty charts and vital statistics - all with a Free Account ← sign up in less than 10 seconds

Question
QuickView
Type Tags Answer
Choices
Curve Question
Difficulty
Psg/Game/S
Difficulty
Explanation
PT15 S3 Q07
+LR
Must be true +MBT
A
1%
148
B
91%
165
C
0%
160
D
4%
159
E
4%
156
130
140
150
+Easier 147.322 +SubsectionMedium

This is a MBT question, since the question stem specifies: The statements above logically commit the politician to which one of the following conclusions?

Our stimulus begins with the conditional indicator unless. If we translate this by negating one of the ideas and making it the sufficient condition, we should get something equivalent to: If our nation does not redistribute wealth, then we cannot alleviate economic injustice AND eventually there will be intolerable economic inequities. This dovetails nicely into the next sentence’s conditional, which tells us that intolerable economic inequities is enough for those who suffer from injustice to resort to violence. Having now given us this set of conditionals about the dire consequences of not redistributing wealth, the politician concludes with a rule; it is our nation’s responsibility to do whatever is necessary to alleviate conditions that lead to violent attempts at social reform.

It should be clear how this rule relates to what we are already told. Based on the preceding two sentences we know that not redistributing wealth means there will be intolerable economic inequities which will in turn necessarily lead to violent attempts at social reform. The lack of redistribution is therefore a condition which necessarily gives rise to violent attempts at social reform, triggering the politicians rule, and consequently it must be true that it is our nation’s responsibility to redistribute wealth.

Answer Choice (A) We haven’t been told anything about whether violent attempts at reform can be justified, we only know that they inevitably arise under certain conditions.

Correct Answer Choice (B) This is exactly what we should have noticed in our pre-phrasing. Whenever the stimulus of a MBT question contains a conditional rule, we should be thinking about whether the sufficient condition is triggered anywhere else in the stimulus.

Answer Choice (C) The stimulus tells us nothing about political expediency and whether it must be chosen over abstract moral principles.

Answer Choice (D) This is consistent with our politician’s statements, but only because they deal with specifically a case that will lead to intolerable social conditions, and mention nothing about what to do if they don’t. We can’t conclude from what the politician says that economic injustice isn’t a problem in other cases.

Answer Choice (E) This is a classic case of confusing necessity for sufficiency. Just because redistribution is required for economic justice does not mean it is enough for economic justice.

Take PrepTest

Review Results

Leave a Reply